Difference between revisions of "2017 AMC 10A Problems/Problem 17"

m (Solution 2)
(Solution 2)
Line 13: Line 13:
 
Quality Control by fasterthanlight
 
Quality Control by fasterthanlight
  
(Note by CarrotKaren: We tried 7, but some might be confused why we concluded that it was the answer after verifying without trying the others, like why wasn't option <math>\textbf{(E)}</math> tried? This is because there can only be one answer in the problem that works, so if we have an option that already works, we can conclude that none of the others work and <math>\textbf{(D)}</math> is the answer.
+
(Note by CarrotKaren: We tried 7, but some might be confused why we concluded that it was the answer after verifying without trying the others, like why wasn't option <math>\textbf{(E)}</math> tried? This is because the problem can only have one correct answer, so if we have an option that already works, we can conclude that none of the others work and <math>\textbf{(D)}</math> is the answer.
  
 
==Video Solution==
 
==Video Solution==

Revision as of 19:49, 23 December 2020

Problem

Distinct points $P$, $Q$, $R$, $S$ lie on the circle $x^{2}+y^{2}=25$ and have integer coordinates. The distances $PQ$ and $RS$ are irrational numbers. What is the greatest possible value of the ratio $\frac{PQ}{RS}$?

$\mathrm{\textbf{(A)}}\ 3\qquad\mathrm{\textbf{(B)}}\ 5\qquad\mathrm{\textbf{(C)}}\ 3\sqrt{5}\qquad\mathrm{\textbf{(D)}}\ 7\qquad\mathrm{\textbf{(E)}}\ 5\sqrt{2}$

Solution 1

Because $P$, $Q$, $R$, and $S$ are lattice points, there are only a few coordinates that actually satisfy the equation. The coordinates are $(\pm 3,\pm 4), (\pm 4, \pm 3), (0,\pm 5),$ and $(\pm 5,0).$ We want to maximize $PQ$ and minimize $RS.$ They also have to be the square root of something, because they are both irrational. The greatest value of $PQ$ happens when $P$ and $Q$ are almost directly across from each other and are in different quadrants. For example, the endpoints of the segment could be $(-4,3)$ and $(3,-4)$ because the two points are almost across from each other. Another possible pair could be $(-4,3)$ and $(5,0)$. To find out which segment is longer, we have to compare the distances from their endpoints to a diameter (which must be the longest possible segment). The closest diameter would be from $(-4,3)$ to $(4,-3)$. The distance between $(3,-4)$ and $(4,-3)$ is shorter than the distance between $(5,0)$ and $(4,-3)$. Therefore, the segment from $(-4,3)$ to $(3,-4)$ is the longest attainable. The least value of $RS$ is when the two endpoints are in the same quadrant and are very close to each other. This can occur when, for example, $R$ is $(3,4)$ and $S$ is $(4,3).$ They are in the same quadrant and no other point on the circle with integer coordinates is closer to the point $(3,4)$ than $(4,3).$ Using the distance formula, we get that $PQ$ is $\sqrt{98}$ and that $RS$ is $\sqrt{2}.$ $\frac{\sqrt{98}}{\sqrt{2}}=\sqrt{49}=\boxed{\mathrm{\textbf{(D)}}\ 7}$

Solution 2

So what we can do is look at the option choices. Since we are aiming for the highest possible ratio, let's try using $7$. Now, looking at the problem alone, we know that to have the largest ratio possible, we have to let $RS$ be the minimum possible value while at the same time using integer coordinates. Thus, the smallest possible value of $RS$ is $\sqrt{1^{2}+1^{2}} = \sqrt{2}$. Assuming that $\frac{PQ}{RS} = 7$, we plug in $RS = \sqrt{2}$ and solve for PQ: $PQ=7\sqrt{2}$. Remember, we don't know if this is possible yet, we are only trying to figure out if it is. But for what values of $x$ and $y$ does $\sqrt{x^{2}+y^{2}}=7\sqrt2$? We see that this can easily be made into a $45-45-90$ triangle. But, instead of substituting $y=x$ into the equation and then using a whole lot of algebra, we can save time and use the little trick, that if in a $45-45-90$ triangle, the two $45$ degree sides have side length $s$, then the hypotenuse is $s\sqrt2$. Using this, we can see that $s=7$, and since our equation does in fact yield a sensible solution, we can be assured that our answer is $\boxed{\mathrm{\textbf{(D)}}\ 7}$.

Quality Control by fasterthanlight

(Note by CarrotKaren: We tried 7, but some might be confused why we concluded that it was the answer after verifying without trying the others, like why wasn't option $\textbf{(E)}$ tried? This is because the problem can only have one correct answer, so if we have an option that already works, we can conclude that none of the others work and $\textbf{(D)}$ is the answer.

Video Solution

https://youtu.be/umr2Aj9ViOA

See Also

2017 AMC 10A (ProblemsAnswer KeyResources)
Preceded by
Problem 16
Followed by
Problem 18
1 2 3 4 5 6 7 8 9 10 11 12 13 14 15 16 17 18 19 20 21 22 23 24 25
All AMC 10 Problems and Solutions

The problems on this page are copyrighted by the Mathematical Association of America's American Mathematics Competitions. AMC logo.png